LSAT and Law School Admissions Forum

Get expert LSAT preparation and law school admissions advice from PowerScore Test Preparation.

User avatar
 Dave Killoran
PowerScore Staff
  • PowerScore Staff
  • Posts: 5853
  • Joined: Mar 25, 2011
|
#90465
Complete Question Explanation
(The complete setup for this game can be found here: lsat/viewtopic.php?f=264&t=32595)

The correct answer choice is (B)

The question stem establishes the following:

G1-Q4-d1.png

Both M and T are inexperienced geologists, so the remaining two selections must be experienced radiobiologists. Thus, two from the group of J, K, and L must be selected. However, due to the third rule, L must be selected, resulting in the following setup:

G1-Q4-d2.png

Accordingly, answer choice (B) is correct.
You do not have the required permissions to view the files attached to this post.
 cboles
  • Posts: 27
  • Joined: Sep 15, 2016
|
#28902
I was stumped when I read this question because of a couple reasons:

1.) How would M and T both be selected for the flight if they are both inexperienced geologists?
2.) Even if they were both selected you can eliminate F and N because they would need to be on the flight and I eliminated P because M and T are already going, so my question is how would you decide between K and L going? Both could go and both don't need to be going so I am stumped.
 cboles
  • Posts: 27
  • Joined: Sep 15, 2016
|
#28903
I now understand my flawed logic in this game. It doesn't say that you have to have one inexperienced and one experienced astronaut from each in order to be valid.
 Nikki Siclunov
PowerScore Staff
  • PowerScore Staff
  • Posts: 1362
  • Joined: Aug 02, 2011
|
#28918
That's exactly right! Good job on figuring this one out on your own :-)

Get the most out of your LSAT Prep Plus subscription.

Analyze and track your performance with our Testing and Analytics Package.